logical reasoning

83
Logical Reasoning Logical Reasoning Section Section

Upload: gurutheboss1

Post on 15-Nov-2014

37 views

Category:

Documents


2 download

TRANSCRIPT

Page 1: Logical Reasoning

Logical Reasoning Logical Reasoning SectionSection

Page 2: Logical Reasoning

ArgumentsArguments

Most logical reasoning questions focus on Most logical reasoning questions focus on arguments, which are sets of statements that arguments, which are sets of statements that present evidence and draw a conclusion on the present evidence and draw a conclusion on the basis of that evidence. These arguments are basis of that evidence. These arguments are generally short and selfgenerally short and self--contained. For example:contained. For example:

Sarah is wellSarah is well--qualified, and the hiring committee is qualified, and the hiring committee is very familiar with her work. Therefore, she will very familiar with her work. Therefore, she will probably receive a job offer. probably receive a job offer.

Page 3: Logical Reasoning

Here, there are two pieces of evidence. These Here, there are two pieces of evidence. These evidence are the premises of the argument. evidence are the premises of the argument.

Sarah is wellSarah is well--qualified.qualified.The hiring committee is very familiar with her work.The hiring committee is very familiar with her work.

The premises are offered in support of the The premises are offered in support of the conclusion, which is:conclusion, which is:

Sarah will probably receive a job offer.Sarah will probably receive a job offer.

Page 4: Logical Reasoning

Identifying the Parts of an ArgumentIdentifying the Parts of an Argument

The most basic parts are the premises and The most basic parts are the premises and conclusions. An argument may even have an conclusions. An argument may even have an intermediate conclusion. For example:intermediate conclusion. For example:

Computer Whiz is a wellComputer Whiz is a well--respected magazine with a respected magazine with a large readership, large readership, so its product endorsements so its product endorsements carry a lot of weight in the computer electronics carry a lot of weight in the computer electronics marketplacemarketplace. The X2000 display monitor was . The X2000 display monitor was recently endorsed by Computer Whiz. It is therefore recently endorsed by Computer Whiz. It is therefore likely that sales of the X2000 monitor will increase likely that sales of the X2000 monitor will increase dramatically.dramatically.

Page 5: Logical Reasoning

Keep in mind that premises and conclusions can come in any Keep in mind that premises and conclusions can come in any order. Premises are presented in support of a conclusion, but torder. Premises are presented in support of a conclusion, but this his does not mean that premises always come before the conclusion. does not mean that premises always come before the conclusion. For example:For example:

Dolores is far more skillful than Victor is at securing the kindDolores is far more skillful than Victor is at securing the kind of financial of financial support the Volunteers for Literacy Program needs, and Dolores dsupport the Volunteers for Literacy Program needs, and Dolores does not oes not have Victorhave Victor’’s propensity for alienating the programs propensity for alienating the program’’s most dedicated s most dedicated volunteers. volunteers. Therefore, the Volunteers for Literacy Program would Therefore, the Volunteers for Literacy Program would benefit if Dolores took Victorbenefit if Dolores took Victor’’s place as director. s place as director. Dolores is fare more skillful than Victor is at securing the kinDolores is fare more skillful than Victor is at securing the kind of financial d of financial support the Volunteers for Literacy Program needs. support the Volunteers for Literacy Program needs. Therefore, the Therefore, the program would benefit if Dolores took Victorprogram would benefit if Dolores took Victor’’s place as directors place as director, , especially since Dolores does not have Victorespecially since Dolores does not have Victor’’s propensity for alienating the s propensity for alienating the programprogram’’s most dedicated volunteers.s most dedicated volunteers.The Volunteers for Literacy Program would benefit if Dolores tooThe Volunteers for Literacy Program would benefit if Dolores took k VictorVictor’’s place as directors place as director, since Dolores is far more skillful than Victor is , since Dolores is far more skillful than Victor is at securing the kind of financial support the program needs and at securing the kind of financial support the program needs and Dolores Dolores does not have Victordoes not have Victor’’s propensity for alienating the programs propensity for alienating the program’’s most s most dedicated volunteers. dedicated volunteers.

Page 6: Logical Reasoning

Keep in mind the presence of indicator words that Keep in mind the presence of indicator words that mark the roles that statements play in arguments. For mark the roles that statements play in arguments. For example, example, ““thereforetherefore”” often precedes a conclusion. often precedes a conclusion. Other common conclusion indicators are Other common conclusion indicators are ““thus,thus,””““hence,hence,”” ““consequently,consequently,”” ““it follows that,it follows that,”” ““it can be it can be concluded that,concluded that,”” and others. Similarly, premises are and others. Similarly, premises are often preceded by indicator words, the most typical often preceded by indicator words, the most typical being being ““sincesince”” and and ““because.because.””But donBut don’’t rely on these. These are just tools to help t rely on these. These are just tools to help you understand arguments. There is no completely you understand arguments. There is no completely mechanical way of identifying the roles that various mechanical way of identifying the roles that various statements play within an argument.statements play within an argument.

Page 7: Logical Reasoning

How the Argument GoesHow the Argument Goes

Once youOnce you’’ve identified the premises and the conclusion, the next ve identified the premises and the conclusion, the next step is to get clear about exactly how the argument is meant to step is to get clear about exactly how the argument is meant to go; that is, how the grounds offered for the conclusion are go; that is, how the grounds offered for the conclusion are actually supposed to bear on the conclusion. In other words, actually supposed to bear on the conclusion. In other words, how are the premises supposed to support the overall how are the premises supposed to support the overall conclusion. conclusion. LetLet’’s consider the argument presented earlier about the s consider the argument presented earlier about the Volunteers for Literacy Program, which concludes that the Volunteers for Literacy Program, which concludes that the program would benefit if Dolores took Victorprogram would benefit if Dolores took Victor’’s place as director. s place as director. Two considerations in support of this conclusion are offered: Two considerations in support of this conclusion are offered: one asserting Doloresone asserting Dolores’’s superiority in securing financial support s superiority in securing financial support and the other charging that Victor is more prone to alienating and the other charging that Victor is more prone to alienating dedicated volunteers. These two considerations are both dedicated volunteers. These two considerations are both relevant to the conclusion since, all other things being equal, relevant to the conclusion since, all other things being equal, a a program benefits from having a director who is both better at program benefits from having a director who is both better at fundfund--raising and less likely to alienate dedicated volunteers.raising and less likely to alienate dedicated volunteers.

Page 8: Logical Reasoning

Questions About How the Argument GoesQuestions About How the Argument Goes

You may encounter questions that ask you about You may encounter questions that ask you about how an argument proceeds overall, or about the how an argument proceeds overall, or about the logical role played by a particular part of an logical role played by a particular part of an argument. Understanding how the relevant argument. Understanding how the relevant argument goes puts you in a position to answer argument goes puts you in a position to answer these questions. Letthese questions. Let’’s look at a couple of s look at a couple of examplesexamples……

Page 9: Logical Reasoning

Example 1Example 1

Red squirrels are known to make holes in the bark of suger mapleRed squirrels are known to make holes in the bark of suger maple trees and to trees and to consume the treesconsume the trees’’ sap. Since sugar maple sap is essentially water with a small sap. Since sugar maple sap is essentially water with a small concentration of sugar, the squirrels almost certainly are afterconcentration of sugar, the squirrels almost certainly are after either water or sugar. either water or sugar. Water is easily available from other sources in places where mapWater is easily available from other sources in places where maple trees grow, so the le trees grow, so the squirrels would not go to the trouble of chewing holes in trees squirrels would not go to the trouble of chewing holes in trees just to get water. just to get water. Therefore, they are probably after the sugar. So how does the aTherefore, they are probably after the sugar. So how does the argument go? rgument go? The conclusion of this argument is easy to identify: red squirreThe conclusion of this argument is easy to identify: red squirrels, in making holes in ls, in making holes in the bark of sugar maple trees, are probably after the sugar contthe bark of sugar maple trees, are probably after the sugar contained in the treesained in the trees’’ sap. sap. The argument arrives at this conclusion by first noting that sinThe argument arrives at this conclusion by first noting that since maple tree sap is ce maple tree sap is essentiall just water and sugar, the squirrels must be after eitessentiall just water and sugar, the squirrels must be after either one or the other. The her one or the other. The argument goes on to reject the idea that it is the water that thargument goes on to reject the idea that it is the water that the squirrels are after, on e squirrels are after, on the grounds that water is readily available for less effort wherthe grounds that water is readily available for less effort where maple trees grow. e maple trees grow. Then, check the answer choices. In this particular case, the beThen, check the answer choices. In this particular case, the best characterization is: st characterization is:

rejecting a possible alternative explanation for an observed pherejecting a possible alternative explanation for an observed phenomenonnomenonThis is not the only way to describe how the argument proceeds, This is not the only way to describe how the argument proceeds, but it is an accurate but it is an accurate characterization and is thus the correct answer. characterization and is thus the correct answer.

Page 10: Logical Reasoning

Example 2Example 2

In order to determine automobile insurance premiums for a driverIn order to determine automobile insurance premiums for a driver, insurance , insurance companies calculate various risk factors; as the risk factors incompanies calculate various risk factors; as the risk factors increase, so does the crease, so does the premium. Certain factors, such as the driverpremium. Certain factors, such as the driver’’s age and past accident history, play s age and past accident history, play an important role in these calculations. Yet these premiums shoan important role in these calculations. Yet these premiums should also increase uld also increase with the frequency with which a person drives. After all, a perwith the frequency with which a person drives. After all, a personson’’s chance of s chance of being involved in a mishap increases in proportion to the numberbeing involved in a mishap increases in proportion to the number of times that of times that person drives. person drives. Question:Question: The claim that insurance premiums should increase as the frequenThe claim that insurance premiums should increase as the frequency with which a cy with which a driver drives increases plays which one of the following roles idriver drives increases plays which one of the following roles in the argument? n the argument? So how does the argument go? First, identify the conclusion. TSo how does the argument go? First, identify the conclusion. To do this, we need o do this, we need to find the position for which the argument offers support. Theto find the position for which the argument offers support. The short phrase short phrase ““after allafter all”” at the beginning of the fourth sentence indicates that the statat the beginning of the fourth sentence indicates that the statement that ement that follows functions as a premise. We also know from the first senfollows functions as a premise. We also know from the first sentence that risk tence that risk factors matter in determining a driverfactors matter in determining a driver’’s automobile insurance premiums. Putting s automobile insurance premiums. Putting all this together, we see that the argument is constructed to suall this together, we see that the argument is constructed to support the position pport the position stated in the third sentence: stated in the third sentence: “…“…these premiums should also increase with the these premiums should also increase with the frequency with which the person drives.frequency with which the person drives.””So the claim that insurance premiums should also increase as theSo the claim that insurance premiums should also increase as the frequency with frequency with which a driver drives increases is the conclusion of the argumenwhich a driver drives increases is the conclusion of the argument. That is its role in t. That is its role in the argument, and the answer choice that expresses this is the cthe argument, and the answer choice that expresses this is the correct one. orrect one.

Page 11: Logical Reasoning

Identifying the Main Conclusion of an ArgumentIdentifying the Main Conclusion of an Argument

Some questions present you with an argument and ask Some questions present you with an argument and ask you to identify ityou to identify it’’s main conclusion. In questions of s main conclusion. In questions of this kind, the conclusion is actually drawn in the this kind, the conclusion is actually drawn in the argument, but it is often stated somewhat indirectly and argument, but it is often stated somewhat indirectly and it is sometimes not signaled by any of the standard it is sometimes not signaled by any of the standard conclusionconclusion--indicator words such as indicator words such as ““thereforetherefore”” or or ““thus.thus.”” To identify the conclusion, therefore, you also To identify the conclusion, therefore, you also need to look at what the statements in the argument need to look at what the statements in the argument mean, and how they are related to each other. Look for mean, and how they are related to each other. Look for a position that the argument has a whole is trying to a position that the argument has a whole is trying to establish, and rule out any statements that, either establish, and rule out any statements that, either directly or indirectly, give reasons for that position. directly or indirectly, give reasons for that position. You should also eliminate statements that merely You should also eliminate statements that merely establish a context or supply background information. establish a context or supply background information.

Page 12: Logical Reasoning

ExampleExample

For example: For example: Journalist: Obviously, though some animals are purely Journalist: Obviously, though some animals are purely carnivorous, none would survive without plants. But the carnivorous, none would survive without plants. But the dependence is mutual. Many plants species would never have dependence is mutual. Many plants species would never have come to be had there been no animals to pollinate, fertilize, come to be had there been no animals to pollinate, fertilize, and broadcast their seeds. Also, plantsand broadcast their seeds. Also, plants’’ photosynthetic photosynthetic activity would deplete the carbon dioxide in Earthactivity would deplete the carbon dioxide in Earth’’s s atmosphere were it not constantly being replenished by the atmosphere were it not constantly being replenished by the exhalation of animals, engine fumes, and smoke from fires, exhalation of animals, engine fumes, and smoke from fires, many set by human beings.many set by human beings.

Question: Question: Which one of the following most accurately expresses Which one of the following most accurately expresses the main conclusion of the journalistthe main conclusion of the journalist’’s argument? s argument?

Page 13: Logical Reasoning

First, read the argument. Are you able to identify the First, read the argument. Are you able to identify the main conclusion by simply reading the argument main conclusion by simply reading the argument through? If notthrough? If not……You may want to go through the argument statement You may want to go through the argument statement by statement and ask about each statement in turn, by statement and ask about each statement in turn, ““Does this statement receive support from some other Does this statement receive support from some other statement?statement?”” If so, the statement is either a subsidiary If so, the statement is either a subsidiary conclusion drawn to support the main conclusion or it conclusion drawn to support the main conclusion or it is itself the main conclusion. If the statement does not is itself the main conclusion. If the statement does not receive support from anything else in the argument, ask receive support from anything else in the argument, ask whether it provides support for some other statement. whether it provides support for some other statement. If it does, itIf it does, it’’s a premise of the argument, and whatever s a premise of the argument, and whatever statement it provides support for is either a main statement it provides support for is either a main conclusion or a subsidiary conclusion. conclusion or a subsidiary conclusion.

Page 14: Logical Reasoning

Matching Patterns of Reasoning in ArgumentsMatching Patterns of Reasoning in Arguments

Questions asking you to match patterns of reasoning in argumentsQuestions asking you to match patterns of reasoning in arguments also also test your ability to determine how an argument goes. It begins test your ability to determine how an argument goes. It begins with an with an argument and then asks you to choose one argument from among theargument and then asks you to choose one argument from among theanswer choices that is most similar in its reasoning to the initanswer choices that is most similar in its reasoning to the initial ial argument. argument. The questions themselves are worded in a variety of ways, includThe questions themselves are worded in a variety of ways, including:ing:

The pattern of reasoning in which of the following arguments is The pattern of reasoning in which of the following arguments is most similar to most similar to that in the argument above? that in the argument above? Which one of the following arguments is most similar in its reasWhich one of the following arguments is most similar in its reasoning to the oning to the argument above?argument above?

These questions are asking for a match in logical structure, thaThese questions are asking for a match in logical structure, that is, the t is, the way the premises fit together to support the conclusion. So, doway the premises fit together to support the conclusion. So, do not not pay any attention to similarity or dissimilarity in subject mattpay any attention to similarity or dissimilarity in subject matter, or to er, or to background material that isnot part of the premises or the conclbackground material that isnot part of the premises or the conclusion.. usion.. Nor should you concern yourself with anything about the particulNor should you concern yourself with anything about the particular ar way the argument is laid out, such as the order in which the preway the argument is laid out, such as the order in which the premises mises and the conclusion are presented. and the conclusion are presented.

Page 15: Logical Reasoning

ExampleExample

All known deposits of the mineral tanzanite are All known deposits of the mineral tanzanite are in Tanzania. Therefore, because Ashley collects in Tanzania. Therefore, because Ashley collects only Tanzanite stones, she is unlikely ever to only Tanzanite stones, she is unlikely ever to collect a stone not originally from Tanzania. collect a stone not originally from Tanzania. The question asks: The question asks: which one of the following is most which one of the following is most similar in its reasoning to the argument above? similar in its reasoning to the argument above?

Page 16: Logical Reasoning

So whatSo what’’s the structure of reasoning in the s the structure of reasoning in the reference argument?reference argument?

There are two premises: one about tanzanite There are two premises: one about tanzanite deposits and one about Ashleydeposits and one about Ashley’’s collecting habits.s collecting habits.There is a conclusion: Ashley is unlikely ever to There is a conclusion: Ashley is unlikely ever to collect a stone not originally from Tanzania. NOTE collect a stone not originally from Tanzania. NOTE that the conclusion says that something is unlikely, that the conclusion says that something is unlikely, not that it will definitely not happen. not that it will definitely not happen. Next step? Check the answer choices and find the Next step? Check the answer choices and find the one with the same pattern of reasoning.one with the same pattern of reasoning.

Page 17: Logical Reasoning

Answer Choice 1: Answer Choice 1: Frogs are the only animals known to live in the lagoon on Frogs are the only animals known to live in the lagoon on Scrag Island. The diet of the owls on Scrag Island consists of Scrag Island. The diet of the owls on Scrag Island consists of nothing but frogs from the island. Therefore, the owls are nothing but frogs from the island. Therefore, the owls are unlikely ever to eat an animal that lives outside of the lagoon.unlikely ever to eat an animal that lives outside of the lagoon.

Whereas the reference argument is strong, this Whereas the reference argument is strong, this argument is seriously flawed. Notice that the two argument is seriously flawed. Notice that the two premises do not rule out the possibility there are frogs premises do not rule out the possibility there are frogs on Scrag Island that do not live in the lagoon. So there on Scrag Island that do not live in the lagoon. So there seems to be a strong possibility that the owls on Scrag seems to be a strong possibility that the owls on Scrag Island eat frogs that arentIsland eat frogs that arent’’ from the lagoon. Hence, the from the lagoon. Hence, the conclusion of this argument receives little or no conclusion of this argument receives little or no support from the premises. If the reasoning in this support from the premises. If the reasoning in this argument were closely parallel to that in the reference argument were closely parallel to that in the reference argument, its premises would provide similary strong argument, its premises would provide similary strong support for its conclusion. So this answer choice is support for its conclusion. So this answer choice is incorrect. incorrect.

Page 18: Logical Reasoning

Answer Choice 2:Answer Choice 2:The only frogs yet discovered on Scrag Island live in the The only frogs yet discovered on Scrag Island live in the lagoon. The diet of all the owls on Scrag Island consists lagoon. The diet of all the owls on Scrag Island consists entirely of frogs on the island, so the owls will probably neverentirely of frogs on the island, so the owls will probably nevereat an animal that lives outside the lagoon. eat an animal that lives outside the lagoon.

Here, the premises provide support for the conclusion Here, the premises provide support for the conclusion in the same way that the premises in the reference in the same way that the premises in the reference argument do for the conclusion of that argument. This argument do for the conclusion of that argument. This argument can be paraphrased in a way that is parallel to argument can be paraphrased in a way that is parallel to the reference argument: All known frogs on Scrag the reference argument: All known frogs on Scrag Island live on the lagoon. Scrag Island owls eat only Island live on the lagoon. Scrag Island owls eat only frogs. It is therefore unlikely that on owl on Scrag frogs. It is therefore unlikely that on owl on Scrag Island will ever eat an animal that does nto live in the Island will ever eat an animal that does nto live in the lagoon. Thus, because the pattern of reasoning in the lagoon. Thus, because the pattern of reasoning in the two arguments is essentially the same, this is the correct two arguments is essentially the same, this is the correct answer choice. answer choice.

Page 19: Logical Reasoning

What Can Be Concluded From the What Can Be Concluded From the Information ProvidedInformation Provided

These questions test your ability to determine These questions test your ability to determine what is supported by a body of available what is supported by a body of available evidence. They ask you to pick one statement evidence. They ask you to pick one statement that can in some way or another be inferred that can in some way or another be inferred from the available evidence. These questions from the available evidence. These questions come in a variety of forms: (1) identifying a come in a variety of forms: (1) identifying a position that is conclusively established by position that is conclusively established by information provided; (2) identifying a position information provided; (2) identifying a position supported by information provided; and (3) supported by information provided; and (3) identifying poinst on which disputants hold identifying poinst on which disputants hold conflicting views.conflicting views.

Page 20: Logical Reasoning

1. Identifying a Position That is Conclusively 1. Identifying a Position That is Conclusively Established by Information ProvidedEstablished by Information Provided

These questions test your ability to identify what follows logicThese questions test your ability to identify what follows logically ally from certain evidence or information. For these questions, you from certain evidence or information. For these questions, you will be presented with information that provides will be presented with information that provides conclusiveconclusivesupport for one of the answer choices. Typical wordings for support for one of the answer choices. Typical wordings for these questions include:these questions include:

If the statements above are true, which one of the following musIf the statements above are true, which one of the following must also be t also be true?true?Which one of the following logically follows from the statementsWhich one of the following logically follows from the statements above?above?

With these questions, you are looking for something that is With these questions, you are looking for something that is guaranteeed to be true by the information you have bene given. guaranteeed to be true by the information you have bene given. That is, the correct answer will be a statement that must be truThat is, the correct answer will be a statement that must be true e if the given information is true. if the given information is true.

Page 21: Logical Reasoning

ExampleExample

Any sale item that is purchased can be returned Any sale item that is purchased can be returned for store credit but not for a refund of the for store credit but not for a refund of the purchase price. Every home appliance and every purchase price. Every home appliance and every piece of gardening equipment is on sale along piece of gardening equipment is on sale along with selected construction tools. with selected construction tools. The question asks:The question asks:

If the statements above are true, which one of the If the statements above are true, which one of the following must also be true?following must also be true?

Page 22: Logical Reasoning

Answer Choice 1:Answer Choice 1:Some construction tools are not returnable for store credit.Some construction tools are not returnable for store credit.

To rule out this answer choice, you need to see that it To rule out this answer choice, you need to see that it does not have to be true if the statements in the passage does not have to be true if the statements in the passage are true. It obviously doesnare true. It obviously doesn’’t have to be true for t have to be true for construction tools that are on sale construction tools that are on sale –– the statements the statements guarantee that those construction tools are returnable guarantee that those construction tools are returnable for store credit. As for the rest of the construction for store credit. As for the rest of the construction tools, those that arentools, those that aren’’t on sale, nothing indicates that t on sale, nothing indicates that they are not returnable for store credit. Based on what they are not returnable for store credit. Based on what the statements say, it is possible, and even likely, that the statements say, it is possible, and even likely, that these tools are returnable for store credit. The answer these tools are returnable for store credit. The answer choice is therefore incorrect. choice is therefore incorrect.

Page 23: Logical Reasoning

Answer Choice 2:Answer Choice 2:No piece of gardening equipment is returnable for a No piece of gardening equipment is returnable for a refund.refund.

We are told that every piece of gardening We are told that every piece of gardening equipment is a sale item and sale items are not equipment is a sale item and sale items are not returnable for a refund. So it must be true that returnable for a refund. So it must be true that gardening equipment is not returnable for a gardening equipment is not returnable for a refund. This is the correct answer.refund. This is the correct answer.

Page 24: Logical Reasoning

Points to ConsiderPoints to Consider

Incorrect answers to questions about what logically follows can Incorrect answers to questions about what logically follows can be be claims that receive some support from the information but that claims that receive some support from the information but that nevertheless could be false even though all of the information inevertheless could be false even though all of the information is s correct. correct. Answer choices are often incorrect because they take things one Answer choices are often incorrect because they take things one step step beyond what the evidence supports. They might make claims that beyond what the evidence supports. They might make claims that are too sweeping; for example, they might say are too sweeping; for example, they might say ““allall”” when the when the evidence supports only a evidence supports only a ““mostmost”” statement. Or where a statement statement. Or where a statement about what about what ““is likely to beis likely to be”” is warranted, an incorrect answer choice is warranted, an incorrect answer choice might say might say ““is.is.”” Or where a statement about Or where a statement about ““all known casesall known cases”” is is warranted, an incorrect answer choice might say warranted, an incorrect answer choice might say ““all cases.all cases.””Also, remember that a modest or limited claim can be a correct Also, remember that a modest or limited claim can be a correct answer even if the information also supports a stronger claim. answer even if the information also supports a stronger claim. If the If the information supports drawing the conclusion that there will be ainformation supports drawing the conclusion that there will be afestival in every month, then it also supports the conclusion thfestival in every month, then it also supports the conclusion that at there will be a festival in June. there will be a festival in June.

Page 25: Logical Reasoning

2. Identifying a Position Supported by 2. Identifying a Position Supported by Information ProvidedInformation Provided

Some questions ask you to identify a position that is supported Some questions ask you to identify a position that is supported by a body of evidence, but not supported conclusively. These by a body of evidence, but not supported conclusively. These questions might be worded as follows:questions might be worded as follows:

Which one of the following is most strongly supported by the Which one of the following is most strongly supported by the information above?information above?Which one of the following can most reasonably be concluded on tWhich one of the following can most reasonably be concluded on the he basis of the information above?basis of the information above?The statements above, if true, most strongly support which one oThe statements above, if true, most strongly support which one of the f the following? following?

For these, you will generally not be presented with an argument,For these, you will generally not be presented with an argument,but merely with some pieces of information. Your task is to but merely with some pieces of information. Your task is to evaluate that information and distinguish between the answer evaluate that information and distinguish between the answer choice that receives strong support from that information (the choice that receives strong support from that information (the correct answer) and answer choices that receive no significant correct answer) and answer choices that receive no significant support (the incorrect answer choices). support (the incorrect answer choices).

Page 26: Logical Reasoning

ExampleExample

Consider the following pieces of information: Consider the following pieces of information: People should avoid taking the antacid calcium People should avoid taking the antacid calcium carbonate in doses larger than half a gram, for carbonate in doses larger than half a gram, for despite its capacity to neutralize stomach acids, despite its capacity to neutralize stomach acids, calcium carbonate can increase the calcium level in calcium carbonate can increase the calcium level in the blood and thus impair kidney function. the blood and thus impair kidney function. Moreover, just half a gram of it can stimulate the Moreover, just half a gram of it can stimulate the production of gastrin, a stomach hormone that production of gastrin, a stomach hormone that triggers acid secretion. triggers acid secretion.

Question: Which one of the following is most Question: Which one of the following is most strongly supported by the information above?strongly supported by the information above?

Page 27: Logical Reasoning

Reading the passage, you find that a certain antacid is Reading the passage, you find that a certain antacid is described as having the obvious intended effect of described as having the obvious intended effect of neutralizing stomach acid but as also having adverse neutralizing stomach acid but as also having adverse side effects if the dosage is too high. One of these side effects if the dosage is too high. One of these adverse effects results in impaired kidney function and adverse effects results in impaired kidney function and other results in acid secretion in the stomach. other results in acid secretion in the stomach. There is a suggestion in the passage that does exceeding There is a suggestion in the passage that does exceeding half a gram are necessary for the first effect to be half a gram are necessary for the first effect to be triggered to any serious extent. The passage also triggered to any serious extent. The passage also suggests that doses of half a gram or more will trigger suggests that doses of half a gram or more will trigger the second effect. the second effect. At this point, itAt this point, it’’s probably a good idea to consider each s probably a good idea to consider each answer choice in turn.answer choice in turn.

Page 28: Logical Reasoning

Answer Choice 1:Answer Choice 1:Doses of calcium carbonate smaller than half a gram can Doses of calcium carbonate smaller than half a gram can reduce stomach acid more effectively than much larger doses reduce stomach acid more effectively than much larger doses do. do.

The passage does give reasons as to why this might be The passage does give reasons as to why this might be true. It tells us that doses of half a gram or more can true. It tells us that doses of half a gram or more can stimulate the production of a stomach hormone that stimulate the production of a stomach hormone that triggers acid secretion. This hormone might counteract triggers acid secretion. This hormone might counteract any extra acidany extra acid--neutralization that comes from additional neutralization that comes from additional calcium carbonate over and above a halfcalcium carbonate over and above a half--gram dose; but gram dose; but then again it might not. Perhaps the extra calcium then again it might not. Perhaps the extra calcium carbonate neutralizes more stomach acid than it carbonate neutralizes more stomach acid than it triggers. For this reason, this answer choice is not triggers. For this reason, this answer choice is not strongly supported by the information. strongly supported by the information.

Page 29: Logical Reasoning

Answer Choice 2:Answer Choice 2:Half a gram of calcium carbonate can causally Half a gram of calcium carbonate can causally contribute to both the secretion and the contribute to both the secretion and the neutralization of stomach acids.neutralization of stomach acids.

How about this choice? Is it supported by the How about this choice? Is it supported by the information? We have noted that at half a gram information? We have noted that at half a gram the secretion of acid in the stomach is triggered. the secretion of acid in the stomach is triggered. The passage mentions the drugThe passage mentions the drug’’s s ““capacity to capacity to neutralize stomach acids,neutralize stomach acids,”” strongly suggesting strongly suggesting that some acidthat some acid--neutralizing effect occurs at any neutralizing effect occurs at any dosage level. So there is strong support in the dosage level. So there is strong support in the passage for both parts of this answer choice. passage for both parts of this answer choice. This is the correct answer choice. This is the correct answer choice.

Page 30: Logical Reasoning

3.3. Identifying Points on Which Disputants Identifying Points on Which Disputants Hold Conflicting ViewsHold Conflicting Views

You may also encounter questions involving two You may also encounter questions involving two speakers where the first speaker puts forward a speakers where the first speaker puts forward a position and the second responds to that position. position and the second responds to that position. You will then be asked something like:You will then be asked something like:

The main point at issue between Sarah and Paul is The main point at issue between Sarah and Paul is whetherwhether……Which one of the following most accurately expresses Which one of the following most accurately expresses the point at issue between Juan and Michiko?the point at issue between Juan and Michiko?On the basis of their statements, Winchell and Trent are On the basis of their statements, Winchell and Trent are committed to disagreeing over whethercommitted to disagreeing over whether……

Page 31: Logical Reasoning

ExampleExample

Mary: Computers will make more information available Mary: Computers will make more information available to ordinary people than was ever available before, thus to ordinary people than was ever available before, thus making it easier for them to acquire knowledge without making it easier for them to acquire knowledge without consulting experts.consulting experts.Joyce: As more knowledge became available in previous Joyce: As more knowledge became available in previous centuries, the need for specialists to synthesize and centuries, the need for specialists to synthesize and explain it to nonspecialists increased. So computers will explain it to nonspecialists increased. So computers will probably create a greater dependency on experts. probably create a greater dependency on experts. Question: Question: The dialogue most strongly supports the claim that The dialogue most strongly supports the claim that Mary and Joyce disagree with each other about whetherMary and Joyce disagree with each other about whether……

Page 32: Logical Reasoning

Answer Choice 1:Answer Choice 1:computers will make more information available to computers will make more information available to ordinary peopleordinary people

Answer Choice 2:Answer Choice 2:dependency on computers will increase with the dependency on computers will increase with the increase of knowledgeincrease of knowledge

Answer Choice 3:Answer Choice 3:computers will increase the need for ordinary people computers will increase the need for ordinary people seeking knowledge to turn to expertsseeking knowledge to turn to experts

And the correct answer isAnd the correct answer is……..Answer Choice 3!Answer Choice 3!

Page 33: Logical Reasoning

Why not 1? Why not 1? Does what Joyce and Mary say show that they Does what Joyce and Mary say show that they disagree about this? disagree about this?

Mary says that computers will make more information Mary says that computers will make more information available to ordinary people. Joyce predicts that available to ordinary people. Joyce predicts that computers will create a greater dependency on experts computers will create a greater dependency on experts because of a historical trend of an increasing dependency because of a historical trend of an increasing dependency on experts whenever more knowledge becomes available on experts whenever more knowledge becomes available to ordinary people. to ordinary people. So Joyce seems to assume that computers will make more So Joyce seems to assume that computers will make more information available to ordinary people. So she probably information available to ordinary people. So she probably agrees with Mary on this point.agrees with Mary on this point.

Page 34: Logical Reasoning

Why not 2?Why not 2?Nothing either Mary or Joyce says commits either of Nothing either Mary or Joyce says commits either of them to a particular view on this position. This is them to a particular view on this position. This is because neither of them explicitly discusses the issue because neither of them explicitly discusses the issue of peopleof people’’s dependency on computers. But there is s dependency on computers. But there is certainly no indication at all that they hold opposing certainly no indication at all that they hold opposing views on whether dependency on computers will views on whether dependency on computers will increase with the increase of knowledge.increase with the increase of knowledge.

Page 35: Logical Reasoning

Why 3?Why 3?Mary straightforwardly disagrees with this claim. Mary straightforwardly disagrees with this claim. Computers, she says, will make it easier for ordinary Computers, she says, will make it easier for ordinary people to acquire knowledge without consulting people to acquire knowledge without consulting experts. Joyce, on the other hand, concludes that experts. Joyce, on the other hand, concludes that computers will create a greater dependency on computers will create a greater dependency on experts. The precedent from past centuries that she experts. The precedent from past centuries that she cites in support of this conclusion makes it clear that cites in support of this conclusion makes it clear that nonspecialists nonspecialists –– that is, ordinary people that is, ordinary people –– will will depend more on experts when knowledge increases. depend more on experts when knowledge increases. So Mary and Joyce disagree on whether the need for So Mary and Joyce disagree on whether the need for ordinary people to turn to experts will be increased ordinary people to turn to experts will be increased by computers. by computers.

Page 36: Logical Reasoning

Necessary Conditions and Sufficient ConditionsNecessary Conditions and Sufficient Conditions

Suppose you read the following statements:Suppose you read the following statements:You donYou don’’t deserve praise for something unless you did it t deserve praise for something unless you did it deliberately.deliberately.Tom deliberately left the door unlocked.Tom deliberately left the door unlocked.

Does it follow from these statements that Tom Does it follow from these statements that Tom deserves praise for leaving the door unlocked?deserves praise for leaving the door unlocked?

No; this doesnNo; this doesn’’t follow. It doesnt follow. It doesn’’t say that any time you do t say that any time you do something deliberately you thereby deserve praise for doing something deliberately you thereby deserve praise for doing it. So the mere fact that Tom did something deliberately is it. So the mere fact that Tom did something deliberately is not enough to bring us to the conclusion that Tom deserves not enough to bring us to the conclusion that Tom deserves praise for doing it. praise for doing it.

Page 37: Logical Reasoning

In technical terms, the first statement expresses a In technical terms, the first statement expresses a necessary necessary conditioncondition. Doing something deliberately is a necessary . Doing something deliberately is a necessary condition for deserving praise for doing it. If the first condition for deserving praise for doing it. If the first statement had said statement had said ““If you do something deliberately then If you do something deliberately then you deserve praise for doing it,you deserve praise for doing it,”” it would be saying that it would be saying that doing something deliberately is a doing something deliberately is a sufficient conditionsufficient condition for for deserving praise for doing it. deserving praise for doing it. Also, the necessary condition just mentioned could have Also, the necessary condition just mentioned could have been stated jut as accurately in several different ways, been stated jut as accurately in several different ways, including: including:

You deserve praise for something only if you did it deliberatelyYou deserve praise for something only if you did it deliberately..You donYou don’’t deserve praise for something if you didnt deserve praise for something if you didn’’t do it t do it deliberately.deliberately.To deserve praise for something, you must have done it To deserve praise for something, you must have done it deliberately.deliberately.

Page 38: Logical Reasoning

Sufficient conditions can also be expressed in several Sufficient conditions can also be expressed in several ways:ways:

If it rains, the sidewalks get wet.If it rains, the sidewalks get wet.Rain is all it takes to get the sidewalks wet.Rain is all it takes to get the sidewalks wet.The sidewalks get wet whenever it rains.The sidewalks get wet whenever it rains.

These statements each tell us that rain is a sufficient These statements each tell us that rain is a sufficient condition for the sidewalks getting wet. It is sufficient, condition for the sidewalks getting wet. It is sufficient, because rain is all that it takes to make the sidewalks because rain is all that it takes to make the sidewalks wet. Notice, however, that these statements do not say wet. Notice, however, that these statements do not say that rain is the only thing that makes the sidewalks wet. that rain is the only thing that makes the sidewalks wet. They do not rule out the possibility that the sidewalks They do not rule out the possibility that the sidewalks can get wet from melting snow or from being sprayed can get wet from melting snow or from being sprayed with a garden hose. So these statements do not express with a garden hose. So these statements do not express necessary conditions for wet sidewalks, only sufficient necessary conditions for wet sidewalks, only sufficient conditions. conditions.

Page 39: Logical Reasoning

So How Do Necessary Conditions Work in Inferences?So How Do Necessary Conditions Work in Inferences?

Suppose the statement reads, Suppose the statement reads, ““You deserve praise for soemthing You deserve praise for soemthing only if you did it deliberately.only if you did it deliberately.”” We are also given a case that satisfies We are also given a case that satisfies the necessary condition, such as the necessary condition, such as ““Tom deliberately left the door Tom deliberately left the door unlocked,unlocked,”” we cannot legitimately draw an inference. Specifically, we cannot legitimately draw an inference. Specifically, the conclusion that Tom deserves praise for leaving the door the conclusion that Tom deserves praise for leaving the door unlocked does not follow. unlocked does not follow. Now suppose that in addition to the first statements above, we aNow suppose that in addition to the first statements above, we are re also told, also told, ““Tom deserves praise for leaving the door unlocked.Tom deserves praise for leaving the door unlocked.”” This This allows us to conclude that Tom deliberately left the door unlockallows us to conclude that Tom deliberately left the door unlocked. ed. Since the first statement says you have to do something deliberaSince the first statement says you have to do something deliberately tely in order to deserve praise for doing it, Tom must have deliberatin order to deserve praise for doing it, Tom must have deliberately ely left the door unlocked if he deserves praise for what he did. left the door unlocked if he deserves praise for what he did. Or, suppose in addition to that first statement, we are told, Or, suppose in addition to that first statement, we are told, ““Tom Tom did not leave the door unlocked deliberately.did not leave the door unlocked deliberately.”” This allows us to This allows us to conclude that Tom does not deserve praise for leaving the door conclude that Tom does not deserve praise for leaving the door unlocked. This follows because the first statement insists thatunlocked. This follows because the first statement insists that only only deliberate actions deserve praise, and we are clearly told that deliberate actions deserve praise, and we are clearly told that TomTom’’s s action is not deliberate. action is not deliberate.

Page 40: Logical Reasoning

How Do Sufficient Conditions Work in Inferences?How Do Sufficient Conditions Work in Inferences?

““If it rains, the sidewalks get wet.If it rains, the sidewalks get wet.””If we are told that the sufficient condition is satisfied (i.e.,If we are told that the sufficient condition is satisfied (i.e.,told that it is raining), then we can legitimately draw the told that it is raining), then we can legitimately draw the inference that the sidewalks are getting wet. inference that the sidewalks are getting wet. Suppose that in addition to the statement above, we are Suppose that in addition to the statement above, we are told that the sidewalks are wet. Can we legitimately told that the sidewalks are wet. Can we legitimately conclude that it rained? conclude that it rained?

No, because the statement does not rule out the possibility thatNo, because the statement does not rule out the possibility thatsomething other than rain can make the sidewalks wet.something other than rain can make the sidewalks wet.

Suppose in addition to the statement above, we are told Suppose in addition to the statement above, we are told that it did not rain. Can we conclude that the sidewalks that it did not rain. Can we conclude that the sidewalks did not get wet? did not get wet?

No, and for the same reasons: the statement does not rule out No, and for the same reasons: the statement does not rule out the possibility that something other than rain can make the the possibility that something other than rain can make the sidewalks wet.sidewalks wet.

Page 41: Logical Reasoning

Understanding the Impact of Additional Understanding the Impact of Additional InformationInformation

The LSAT typically includes several questions The LSAT typically includes several questions that test your ability to see how additional facts that test your ability to see how additional facts bear on an argument. Typical wordings of such bear on an argument. Typical wordings of such questions are:questions are:

Which one of the following, if true, most Which one of the following, if true, most strengthens the argument?strengthens the argument?Which one of the following, if true, most weakens Which one of the following, if true, most weakens the argument?the argument?In order to evaluate the argument, which one of the In order to evaluate the argument, which one of the following would it be most useful to determine?following would it be most useful to determine?

Page 42: Logical Reasoning

ExampleExample

A survey of oilA survey of oil--refinery workers who work with MBTE, an refinery workers who work with MBTE, an ingredient currently used in some smogingredient currently used in some smog--reducing gasolines, found reducing gasolines, found an alarming incidence of complaints about headaches, fatigue, anan alarming incidence of complaints about headaches, fatigue, and d shortness of breath. Since gasoline containing MBTE will soon shortness of breath. Since gasoline containing MBTE will soon be widely used, we can expect an increased incidence of headachebe widely used, we can expect an increased incidence of headaches, s, fatigue, and shortness of breath. fatigue, and shortness of breath. Additional information could, depending on what this Additional information could, depending on what this information is, make the argument presented here information is, make the argument presented here stronger or weaker. stronger or weaker.

Page 43: Logical Reasoning

For example, suppose it is true that most oilFor example, suppose it is true that most oil--refinery refinery workers who do not work with MBTE also have a very workers who do not work with MBTE also have a very high incidence of headaches, fatigue, and shortness of high incidence of headaches, fatigue, and shortness of breath. This would provide evidence that it is not breath. This would provide evidence that it is not MBTE but some other factor that is primarily MBTE but some other factor that is primarily responsible for these symptoms. But if we have responsible for these symptoms. But if we have evidence that something other than MBTE is causing evidence that something other than MBTE is causing these symptoms, then the argument provides only very these symptoms, then the argument provides only very weak support, if any, for its conclusion. That is, the weak support, if any, for its conclusion. That is, the argumentargument’’s original premises, when combined with the s original premises, when combined with the additional information, make a much weaker case for additional information, make a much weaker case for the argumentthe argument’’s conclusion than those premises did s conclusion than those premises did alone. In other words, the new information has made alone. In other words, the new information has made the argument weaker. the argument weaker.

Page 44: Logical Reasoning

Suppose, now, that gasoline containing MBTE Suppose, now, that gasoline containing MBTE has already been introduced in a few has already been introduced in a few metropolitan areas, and since it was first metropolitan areas, and since it was first introduced, those areas have reported increased introduced, those areas have reported increased complaints about headaches, fatigue, and complaints about headaches, fatigue, and shortness of breath. This would provide shortness of breath. This would provide evidence that when MBTE is used as a gasoline evidence that when MBTE is used as a gasoline additive, it increases the incidence of these additive, it increases the incidence of these symptoms not just among refinery workers who symptoms not just among refinery workers who work closely with it but also among the general work closely with it but also among the general public. So we now have evidence that is more public. So we now have evidence that is more directly relevant to the argumentdirectly relevant to the argument’’s conclusion. s conclusion. Thus, the new evidence has made the argument Thus, the new evidence has made the argument stronger. stronger.

Page 45: Logical Reasoning

Example 2Example 2

A recent study reveals that television advertising does not A recent study reveals that television advertising does not significantly affect childrensignificantly affect children’’s preferences for breakfast s preferences for breakfast cereals. The study compared two groups of children. One cereals. The study compared two groups of children. One group had watched no television, and the other group had group had watched no television, and the other group had watched average amounts of television and its advertising. watched average amounts of television and its advertising. Both groups strongly preferred the sugary cereals heavily Both groups strongly preferred the sugary cereals heavily advertised on television. advertised on television.

Page 46: Logical Reasoning

WhatWhat’’s the conclusion of this argument?s the conclusion of this argument?That television advertising does not significantly affect That television advertising does not significantly affect childrenchildren’’s preferences for breakfast cereals. s preferences for breakfast cereals.

QuestionQuestion: : Which one of the following statements, if true, most Which one of the following statements, if true, most weakens the argument?weakens the argument?Answer Choice 1Answer Choice 1: Most of the children in the group : Most of the children in the group that had watched television were already familiar with that had watched television were already familiar with the advertisements for these cereals. the advertisements for these cereals. Answer Choice 2Answer Choice 2: Both groups rejected cereals low in : Both groups rejected cereals low in sugar even when these cereals were heavily advertised sugar even when these cereals were heavily advertised on television.on television.Answer Choice 3Answer Choice 3: The preferences of children who do : The preferences of children who do not watch television advertising are influenced by the not watch television advertising are influenced by the preferences of children who watch television preferences of children who watch television advertising. advertising.

Page 47: Logical Reasoning

LetLet’’s consider Answer Choice 1s consider Answer Choice 1This information suggests that even if the television This information suggests that even if the television advertising influenced the preferences of the advertising influenced the preferences of the children who watched television, this influence children who watched television, this influence occurred some time ago. But this does not really occurred some time ago. But this does not really imply anything about whether the advertising did imply anything about whether the advertising did influence the childreninfluence the children’’s preferences. So the s preferences. So the information provided by this answer choice neither information provided by this answer choice neither strenghtens nor weakens the argument. strenghtens nor weakens the argument.

LetLet’’s consider Answer Choice 2s consider Answer Choice 2This information provides additional evidence in This information provides additional evidence in favor of the argumentfavor of the argument’’s conclusion that television s conclusion that television advertising does not significantly affect childrenadvertising does not significantly affect children’’s s cereal preferences. So this argument strenghtens the cereal preferences. So this argument strenghtens the argument rather than weakens it.argument rather than weakens it.

Page 48: Logical Reasoning

LetLet’’s look at Answer Choice 3s look at Answer Choice 3The reason originally offered for the conclusion is that the The reason originally offered for the conclusion is that the two groups of children do not differ in their preferences. But two groups of children do not differ in their preferences. But if the preferences of the children who do not watch television if the preferences of the children who do not watch television advertising are influenced by the preferences of those who do advertising are influenced by the preferences of those who do watch it, then the fact that the two groups do not differ in watch it, then the fact that the two groups do not differ in their preferences provides little, if any, reason to think that their preferences provides little, if any, reason to think that none of the childrennone of the children’’s preferences were affected by television s preferences were affected by television advertising. After all, it could well be that the preferences oadvertising. After all, it could well be that the preferences of f the children who watched television were strongly influenced the children who watched television were strongly influenced by the advertising, and these childrenby the advertising, and these children’’s preferences in turn s preferences in turn strongly influenced the preferences of those who did not strongly influenced the preferences of those who did not watch television, with the result that the two groups had the watch television, with the result that the two groups had the same preferences. So when combined with the additional same preferences. So when combined with the additional information, the argumentinformation, the argument’’s original premises make a much s original premises make a much weaker case for the argumentweaker case for the argument’’s conclusion than they did s conclusion than they did alone. Thus, this is the correct answer. alone. Thus, this is the correct answer.

Page 49: Logical Reasoning

AssumptionsAssumptionsThe Logical Reasoning section typically includes several questioThe Logical Reasoning section typically includes several questions ns that test your ability to identify assumptions of arguments. Anthat test your ability to identify assumptions of arguments. Anassumption of an argument plays a role in establishing the assumption of an argument plays a role in establishing the conclusion. However, unlike a premise, an assumption is not conclusion. However, unlike a premise, an assumption is not something that the arguer explicitly asserts to be true; an assusomething that the arguer explicitly asserts to be true; an assumption mption is instead just treated as true for purposes of the argument. is instead just treated as true for purposes of the argument. Logical Reasoning questions ask only about unstated (or tacit) Logical Reasoning questions ask only about unstated (or tacit) assumptions. Unstated assumptions can figure only in arguments assumptions. Unstated assumptions can figure only in arguments that are not entirely complete, that is, in arguments in which sthat are not entirely complete, that is, in arguments in which some of ome of the things required to establish the conclusion are left unstatethe things required to establish the conclusion are left unstated. d. An assumption is a sufficient one if adding it to the argumentAn assumption is a sufficient one if adding it to the argument’’s s premises would produce a conclusive argument, that is, an argumepremises would produce a conclusive argument, that is, an argument nt with no gaps in its support for the conclusion. An assumption iwith no gaps in its support for the conclusion. An assumption is a s a necessary one if it is something that must be true in order for necessary one if it is something that must be true in order for the the argument to succeed. An assumption can be both necessary and argument to succeed. An assumption can be both necessary and sufficient. sufficient.

Page 50: Logical Reasoning

Sufficient AssumptionsSufficient AssumptionsTypical wordings of questions that ask you to identify Typical wordings of questions that ask you to identify sufficient assumptions are:sufficient assumptions are:

Which one of the following, if assumed, enables the Which one of the following, if assumed, enables the conclusion of the argument to be properly drawn?conclusion of the argument to be properly drawn?The conclusion follows logically from the premises if which The conclusion follows logically from the premises if which one of the following is assumed?one of the following is assumed?

LetLet’’s look at an example: s look at an example: Prompt: Vague laws set vague limits on peoplePrompt: Vague laws set vague limits on people’’s freedom, s freedom, which makes it impossible for them to know for certain which makes it impossible for them to know for certain whether their actions are legal. Thus, under vague laws whether their actions are legal. Thus, under vague laws people cannot feel secure. people cannot feel secure. Question: Question: The conclusion follows logically if which one of the following The conclusion follows logically if which one of the following is assumed?is assumed?

Page 51: Logical Reasoning

LetLet’’s first identify the conclusion of the s first identify the conclusion of the argument and the premises offered in its argument and the premises offered in its support. support.

The conclusion is signaled by the conclusion The conclusion is signaled by the conclusion indicator indicator ““thusthus”” and reads and reads “…“…under vague laws under vague laws people cannot feel secure.people cannot feel secure.””There are two considerations explicitly presented in There are two considerations explicitly presented in support of this conclusion. support of this conclusion.

First, that vague laws set vague limits on peopleFirst, that vague laws set vague limits on people’’s s freedom.freedom.Second, that having vague limits set on their freedom Second, that having vague limits set on their freedom makes it impossible for people to know for certain makes it impossible for people to know for certain whether their actions are legal. whether their actions are legal.

Page 52: Logical Reasoning

Note that the premises, though they tell us Note that the premises, though they tell us certain things about vague laws, make no explicit certain things about vague laws, make no explicit reference to whether people feel secure, and not reference to whether people feel secure, and not feeling secure is what the conclusion is about. feeling secure is what the conclusion is about. For the conclusion to follow logically, this gap For the conclusion to follow logically, this gap has to be bridged. So lethas to be bridged. So let’’s look at the answer s look at the answer choices: choices:

(a) People can feel secure only if they know for (a) People can feel secure only if they know for certain whether their actions are legal.certain whether their actions are legal.(b) If people know for certain whether their actions (b) If people know for certain whether their actions are legal, they can feel secure. are legal, they can feel secure.

Page 53: Logical Reasoning

Your task is to identify the answer choice that, together with tYour task is to identify the answer choice that, together with the he premises youpremises you’’ve been given, will provide conclusive support for the ve been given, will provide conclusive support for the conclusion. conclusion. LetLet’’s look at (a):s look at (a):

The explicit premises of the argument tell you that under vague The explicit premises of the argument tell you that under vague laws people laws people cannot know for certain whether their actions are legal. (a) tecannot know for certain whether their actions are legal. (a) tells you that if lls you that if people do not know for certain whether their actions are legal, people do not know for certain whether their actions are legal, they cannot they cannot feel secure. So putting the explicit premises and (a) together,feel secure. So putting the explicit premises and (a) together, you can infer you can infer that under vague laws people cannot feel secure. So, the concluthat under vague laws people cannot feel secure. So, the conclusion follows sion follows logically if (a) is assumed!logically if (a) is assumed!

LetLet’’s look at (b):s look at (b):(b) tells us about one circumstance in which people can feel sec(b) tells us about one circumstance in which people can feel secure. ure. However, the argumentHowever, the argument’’s conclusion will not follow logically without the s conclusion will not follow logically without the right kind of information about the circumstances in which peoplright kind of information about the circumstances in which people cannot e cannot feel secure. (b) does not give us any such information directlyfeel secure. (b) does not give us any such information directly. Moreover, we . Moreover, we cannot infer such information from what (b) does tell us. Thus,cannot infer such information from what (b) does tell us. Thus, since (b) tells since (b) tells us nothing about circumstances in which people cannot feel securus nothing about circumstances in which people cannot feel secure, it has e, it has nothing to contribute to reaching the argumentnothing to contribute to reaching the argument’’s conclusion that people s conclusion that people cannot feel secure under vague laws. cannot feel secure under vague laws.

Page 54: Logical Reasoning

Necessary AssumptionsNecessary Assumptions

Typical wordings of questions that ask you to identify Typical wordings of questions that ask you to identify necessary assumptions include the following: necessary assumptions include the following:

The argument relies on assuming which one of the following?The argument relies on assuming which one of the following?The argument depends on the assumption thatThe argument depends on the assumption thatWhich one of the following is an assumption required by the Which one of the following is an assumption required by the argument?argument?

In these cases, you will find that there is at least one In these cases, you will find that there is at least one significant gap in the argument. A necessary significant gap in the argument. A necessary assumption, therefore, is an indispensable link in the assumption, therefore, is an indispensable link in the support for the conclusion of an argument. support for the conclusion of an argument.

Page 55: Logical Reasoning

ExampleExample

Since Mayor Drabble always repays her political debts Since Mayor Drabble always repays her political debts as soon as possible, she will almost certainly appoint as soon as possible, she will almost certainly appoint Lee to be the new head of its arts commission. Lee has Lee to be the new head of its arts commission. Lee has wanted that job for a long time, and Drabble owes Lee wanted that job for a long time, and Drabble owes Lee a lot for his support in the last election.a lot for his support in the last election.In order to show that Lee is the likely appointee, there In order to show that Lee is the likely appointee, there cancan’’t be anyone else to whom Drabble has owed a such t be anyone else to whom Drabble has owed a such a large and longa large and long--standing political debt and for whom standing political debt and for whom this appointment would be adequate repayment. This this appointment would be adequate repayment. This idea of there being no one ahead of Lee in line is the idea of there being no one ahead of Lee in line is the sort of unstated but indispensable link in the support sort of unstated but indispensable link in the support for the conclusion that we mean when we speak of a for the conclusion that we mean when we speak of a necessary assumption of an argument.necessary assumption of an argument.

Page 56: Logical Reasoning

A Useful TestA Useful Test

To see whether an answer choice is a necessary To see whether an answer choice is a necessary assumption, suppose that what is stated in that assumption, suppose that what is stated in that answer choice is false. If under those answer choice is false. If under those circumstances the premises of the argument fail circumstances the premises of the argument fail to support the conclusion, the answer choice to support the conclusion, the answer choice being evaluated is a necessary assumption.being evaluated is a necessary assumption.

Page 57: Logical Reasoning

LetLet’’s Test the Tests Test the Test

Advertisement: Attention pond owners! NinetyAdvertisement: Attention pond owners! Ninety--eight eight percent of mosquito larvae in a pond die within percent of mosquito larvae in a pond die within minutes after the pond has been treated with BTI. Yet minutes after the pond has been treated with BTI. Yet BTI is not toxic to fish, birds, animals, plants, or BTI is not toxic to fish, birds, animals, plants, or beneficial insects. So by using BTI regularly to destroy beneficial insects. So by using BTI regularly to destroy their larvae, you can greatly reduce populations of their larvae, you can greatly reduce populations of pesky mosquitoes that hatch in your pond, and can do pesky mosquitoes that hatch in your pond, and can do so without diminishing the populations of fish, frogs, or so without diminishing the populations of fish, frogs, or beneficial insects in and around the pond. beneficial insects in and around the pond. Question: Question: Which one of the following is an assumption on Which one of the following is an assumption on which the argument depends? which the argument depends?

Page 58: Logical Reasoning

Before proceeding to the answer choices, letBefore proceeding to the answer choices, let’’s turn to s turn to the prompt first. The conclusion is that regular the prompt first. The conclusion is that regular applications of BTI in a pond can, without reducing applications of BTI in a pond can, without reducing populations of assorted pond life, greatly reduce the populations of assorted pond life, greatly reduce the numbers of mosquitoes that emerge from the pond. numbers of mosquitoes that emerge from the pond. The evidence is that BTI kills almost all of the The evidence is that BTI kills almost all of the mosquito larvae in the pond, but does not kill (or even mosquito larvae in the pond, but does not kill (or even harm) other pond life.harm) other pond life.Now letNow let’’s turn to the answer choices:s turn to the answer choices:

(1) The most effective way to control the number of (1) The most effective way to control the number of mosquitoes in a given area is to destroy the mosquito larvae mosquitoes in a given area is to destroy the mosquito larvae in that area.in that area.(2) The fish, frogs, and beneficial insects in and around a (2) The fish, frogs, and beneficial insects in and around a pondpond--ownerowner’’s pond do not depend on mosquito larvae as an s pond do not depend on mosquito larvae as an important source of food. important source of food.

Page 59: Logical Reasoning

LetLet’’s apply the test to answer choice (1) by asking s apply the test to answer choice (1) by asking whether the argument would fail if this answer choice whether the argument would fail if this answer choice were false. That is, would the argument fail if the were false. That is, would the argument fail if the destruction of mosquito larvae were not the most destruction of mosquito larvae were not the most effective way to control the numbers of mosquitoes? effective way to control the numbers of mosquitoes? Definitely not. Definitely not. LetLet’’s apply the test to answer choice (2) by asking s apply the test to answer choice (2) by asking whether the argument would fail it this answer choice whether the argument would fail it this answer choice were false. That is, would the argument fail if these were false. That is, would the argument fail if these creatures did depend on mosquito larvae for food? Yes creatures did depend on mosquito larvae for food? Yes it would; after all, if the use of BTI means that fish, it would; after all, if the use of BTI means that fish, frogs, and so forth will be deprived of a food that is frogs, and so forth will be deprived of a food that is important to them (mosquito larvae), then there is no important to them (mosquito larvae), then there is no reason to conclude that these creatures will survive in reason to conclude that these creatures will survive in undiminished numbers. So denying this answer choice undiminished numbers. So denying this answer choice would cause the argument to fail; we have found a would cause the argument to fail; we have found a necessary assumption. necessary assumption.

Page 60: Logical Reasoning

PrinciplesPrinciples

Some Logical Reasoning questions test your ability to Some Logical Reasoning questions test your ability to apply general rules and principles and to understand apply general rules and principles and to understand their use. their use. Principles are broad guidelines concerning what kinds Principles are broad guidelines concerning what kinds of actions, judgments, policies, and so on are of actions, judgments, policies, and so on are appropriate. appropriate. You may see several kinds of questions involving You may see several kinds of questions involving principles. You may be given a principle and be asked principles. You may be given a principle and be asked which action conforms to it, or which judgment it which action conforms to it, or which judgment it justifies, or which argument relies on it. Alternatively, justifies, or which argument relies on it. Alternatively, the question may present a judgment, decision, or the question may present a judgment, decision, or argument and ask which principle is appealed to in argument and ask which principle is appealed to in making that judgment, decision, or argument. making that judgment, decision, or argument.

Page 61: Logical Reasoning

Applying a Principle That Is GivenApplying a Principle That Is GivenExample (this is the principle):Example (this is the principle):

People who receive unsolicited advice from someone whose advantaPeople who receive unsolicited advice from someone whose advantage ge would be served if that advice is taken should regard the proffewould be served if that advice is taken should regard the proffered advice red advice with skepticism unless there is good reason to think that their with skepticism unless there is good reason to think that their interests interests substantially coincide with those of the advice giver in the cirsubstantially coincide with those of the advice giver in the circumstances cumstances in question. in question.

Question (the question refers to the principle):Question (the question refers to the principle):This principle, if accepted, would justify which one of the follThis principle, if accepted, would justify which one of the following owing judgments?judgments?

Correct Answer:Correct Answer:While shopping for a refrigerator, Razmig is approached by a salWhile shopping for a refrigerator, Razmig is approached by a salesperson esperson who, on the basis of her personal experience, warns him against who, on the basis of her personal experience, warns him against the least the least expensive model. However, the salespersonexpensive model. However, the salesperson’’s commission increases with s commission increases with the price of the refrigerator sold, so Razmig should not reject the price of the refrigerator sold, so Razmig should not reject the least the least expensive model on the salespersonexpensive model on the salesperson’’s advice alone. s advice alone.

Page 62: Logical Reasoning

The first question you should ask is: The first question you should ask is: ““Does someone in this situation receive Does someone in this situation receive unsolicited advice from someone whose advantage would be served unsolicited advice from someone whose advantage would be served if that advice is if that advice is taken?taken?”” If the answer is If the answer is ““yes,yes,”” then the case under consideration falls within the then the case under consideration falls within the range of situations to which the principle applies. If the answrange of situations to which the principle applies. If the answer is er is ““no,no,”” then the then the principle offers no guidance. Here, someone principle offers no guidance. Here, someone –– Razmig Razmig –– does receive advice. If does receive advice. If Razmig took the advice, this would be to the advantage of the adRazmig took the advice, this would be to the advantage of the advice giver (the vice giver (the salesperson), because the salesperson would receive a higher comsalesperson), because the salesperson would receive a higher commission than she mission than she would otherwise. And notice that the advice is in fact unsoliciwould otherwise. And notice that the advice is in fact unsolicited, because the ted, because the salesperson approached Razmig without his asking for help. salesperson approached Razmig without his asking for help. The next question you should ask is, The next question you should ask is, ““Does the situation culminate in a judgment Does the situation culminate in a judgment that the advice should be regarded with skepticim?that the advice should be regarded with skepticim?”” The answer is again The answer is again ““yes.yes.”” The The judgment that Razmig should not reject the least expensive modeljudgment that Razmig should not reject the least expensive model solely on the solely on the salespersonsalesperson’’s advice is a judgment that treats the advice given s advice is a judgment that treats the advice given –– to avoid buying the to avoid buying the least expensive model least expensive model –– skeptically. skeptically. ThereThere’’s one last step! The principle restricts itself to situations is one last step! The principle restricts itself to situations in which the person n which the person giving the advice and the person receiving the advice do not havgiving the advice and the person receiving the advice do not have interests that e interests that coincide. So you need to ask one more question: coincide. So you need to ask one more question: ““Is there reason to think that the Is there reason to think that the interests of Razmig and those of the salesperson substantially cinterests of Razmig and those of the salesperson substantially coincide in this oincide in this matter?matter?”” Since Razmig probably wants to spend no more than he has to anSince Razmig probably wants to spend no more than he has to and since d since the salesperson probably wants Razmig to spend freely, there is the salesperson probably wants Razmig to spend freely, there is reason to think that reason to think that in this matter their interests do in this matter their interests do notnot coincide. So the principle applies to the situation coincide. So the principle applies to the situation and justifies the judgment. and justifies the judgment.

Page 63: Logical Reasoning

Identifying a Guiding PrincipleIdentifying a Guiding Principle

Example: Example: Marianne is a professional chess player who hums audibly while pMarianne is a professional chess player who hums audibly while playing laying her matches, thereby distracting her opponents. When ordered byher matches, thereby distracting her opponents. When ordered by chess chess officials to cease humming or else be disqualified from professiofficials to cease humming or else be disqualified from professional chess, onal chess, Marianne protested the order. She argued that since she was unaMarianne protested the order. She argued that since she was unaware of ware of her humming, her humming was involuntary and that therefore she her humming, her humming was involuntary and that therefore she should not be held responsible for it.should not be held responsible for it.

Question:Question:Which one of the following principles, if valid, most helps to sWhich one of the following principles, if valid, most helps to support upport MarianneMarianne’’s argument against the order?s argument against the order?

In answering this question, you need to compare the specific In answering this question, you need to compare the specific circumstances presented in the passage with the principle circumstances presented in the passage with the principle presented in each answer choice.presented in each answer choice.

Page 64: Logical Reasoning

Answer Choices:Answer Choices:(a) Of a player(a) Of a player’’s actions, only those that are s actions, only those that are voluntary should be used as justification for voluntary should be used as justification for disqualifying that player from professional chess.disqualifying that player from professional chess.(b) Chess players should be disqualified from (b) Chess players should be disqualified from professional chess matches if they regularly attempt professional chess matches if they regularly attempt to distract their opponents. to distract their opponents.

And the correct answer choice is:And the correct answer choice is:Answer choice (a)! Answer choice (a)! But why? But why?

Page 65: Logical Reasoning

Consider answer choice (a): Consider answer choice (a): Does the principle expressed in (a) apply to MarianneDoes the principle expressed in (a) apply to Marianne’’s situation? It s situation? It is clear that the principle concerns which of a chess playeris clear that the principle concerns which of a chess player’’s actions s actions can appropriately be used as justification for disqualifying thacan appropriately be used as justification for disqualifying that player t player from professional chess. Since the argument in the passage is from professional chess. Since the argument in the passage is concerned with whether one of Marianneconcerned with whether one of Marianne’’s actions s actions –– humming while humming while playing playing –– should disqualify her from professional chess, it definitely should disqualify her from professional chess, it definitely falls under the range of situations to which the principle applifalls under the range of situations to which the principle applies. es. The principle will help support MarianneThe principle will help support Marianne’’s argument if it leads to a s argument if it leads to a judgment that Mariannejudgment that Marianne’’s humming while playing should not be used s humming while playing should not be used as justification for disqualifying her from playing. According as justification for disqualifying her from playing. According to a to a subsidiary conclusion of Mariannesubsidiary conclusion of Marianne’’s argument, her humming is s argument, her humming is involuntary (this is supported by the claim that she was unawareinvoluntary (this is supported by the claim that she was unaware of of it). The principle asserts that only voluntary actions should bit). The principle asserts that only voluntary actions should be used e used as justification for disqualifying a player from professional chas justification for disqualifying a player from professional chess, so ess, so this principle, together with the subsidiary conclusion of Mariathis principle, together with the subsidiary conclusion of Mariannenne’’s s argument, leads to the judgment that Marianneargument, leads to the judgment that Marianne’’s humming should s humming should not be used as justification for disqualifying her. Thus the prnot be used as justification for disqualifying her. Thus the principle inciple does help support Mariannedoes help support Marianne’’s argument. s argument.

Page 66: Logical Reasoning

Consider answer choice (b):Consider answer choice (b):Does this principle apply to MarianneDoes this principle apply to Marianne’’s situation? Yes, it s situation? Yes, it apparently does since it is also about the conditions under whicapparently does since it is also about the conditions under which h chess players should be disqualified from professional chess chess players should be disqualified from professional chess matches. So now we ask whether the principle establishes, or matches. So now we ask whether the principle establishes, or helps establish, the conclusion that Marianne should not be helps establish, the conclusion that Marianne should not be disqualified for humming during matches? The answer is no. disqualified for humming during matches? The answer is no. This principle just gives one condition under which a chess playThis principle just gives one condition under which a chess player er should be disqualified should be disqualified –– when the player regularly attempts to when the player regularly attempts to distract opponents. Since Mariannedistract opponents. Since Marianne’’s humming is, she argues, s humming is, she argues, involuntary, we can conclude that she is not trying to distract involuntary, we can conclude that she is not trying to distract her her opponents. Thus the principle does not lead to the judgment opponents. Thus the principle does not lead to the judgment that Marianne should be disqualified for humming. But this doesthat Marianne should be disqualified for humming. But this doesnot mean that the principle leads to the judgment that Marianna not mean that the principle leads to the judgment that Marianna should not be disqualified. After all, it is compatible with thshould not be disqualified. After all, it is compatible with the e principle that there are other conditions under which a player principle that there are other conditions under which a player should be disqualified, and such conditions could include should be disqualified, and such conditions could include humming while playing. So the principle does not lead to the humming while playing. So the principle does not lead to the conclusion that Marianne should not be disqualified from conclusion that Marianne should not be disqualified from professional matches and thus does not provide any support for professional matches and thus does not provide any support for MarianneMarianne’’s argument. s argument.

Page 67: Logical Reasoning

Flaws in Arguments: Flaws in Arguments: Identifying Argument FlawsIdentifying Argument Flaws

The Logical Reasoning section also includes a number of questionThe Logical Reasoning section also includes a number of questions that s that ask youto identify a flaw of reasoning that has been committed iask youto identify a flaw of reasoning that has been committed in an n an argument. argument. Questions of this kind can be worded in a variety of ways. For Questions of this kind can be worded in a variety of ways. For example:example:

The reasoning in the argument is flawed because the argumentThe reasoning in the argument is flawed because the argument……The argument commits which one of the following errors of reasonThe argument commits which one of the following errors of reasoning? ing? The argumentThe argument’’s reasoning is questionable because the argument fails to rule os reasoning is questionable because the argument fails to rule out the ut the possibility thatpossibility that……The reasoning above is most vulnerable to criticism on the grounThe reasoning above is most vulnerable to criticism on the grounds that itds that it……

Test questions about flawed reasoning require you to recognize iTest questions about flawed reasoning require you to recognize in what n what way an argument is defective in its reasoning. They will not reway an argument is defective in its reasoning. They will not require you quire you to decide whether or not the argument is flawed. That judgment to decide whether or not the argument is flawed. That judgment is is already made and is expressed in the wording of the question. Yalready made and is expressed in the wording of the question. Your task our task is to recognize which one of the answer choices describes an erris to recognize which one of the answer choices describes an error of or of reasoning that the argument makes. reasoning that the argument makes.

Page 68: Logical Reasoning

Example 1:Example 1:Consider the following brief exchage: Consider the following brief exchage:

PhysicistPhysicist: The claim that low: The claim that low--temperate nuclear fusion can be temperate nuclear fusion can be achieved entirely by chemical means is based on chemical achieved entirely by chemical means is based on chemical experiments in which the measurements and calculations are experiments in which the measurements and calculations are inaccurate. inaccurate. ChemistChemist: But your challenge is ineffectual, since you are : But your challenge is ineffectual, since you are simply jealous at the thought that chemists might have solved simply jealous at the thought that chemists might have solved a problem that physicists have been unable to solve. a problem that physicists have been unable to solve.

Question: Question: Which one of the following is the strongest criticism of Which one of the following is the strongest criticism of the chemistthe chemist’’s response to the physicists response to the physicist’’s challenge? s challenge?

Page 69: Logical Reasoning

Before proceeding to the answer choices, quickly Before proceeding to the answer choices, quickly consider what appears to be wrong with the chemistconsider what appears to be wrong with the chemist’’s s response. Notice that the chemist claims that the response. Notice that the chemist claims that the physicistphysicist’’s challenge is ineffectual but doesns challenge is ineffectual but doesn’’t actually t actually engage the substance of the physicistengage the substance of the physicist’’s challenge. s challenge. Instead, the chemist accuses the physicist of Instead, the chemist accuses the physicist of professional jealousy and dismisses the physicistprofessional jealousy and dismisses the physicist’’s s challenge purely on that basis. But there is no reason to challenge purely on that basis. But there is no reason to think that a challenge, even if it is fueld by jealousy, think that a challenge, even if it is fueld by jealousy, cannot be on target. cannot be on target. Now with this in mind, letNow with this in mind, let’’s consider the answer choice:s consider the answer choice:

(a) It fails to establish that perfect accuracy of measurements (a) It fails to establish that perfect accuracy of measurements and calculation is possible.and calculation is possible.(b) It is directed against the proponent of a claim rather than (b) It is directed against the proponent of a claim rather than against the claim itself. against the claim itself.

Page 70: Logical Reasoning

Consider answer choice (a):Consider answer choice (a):This statement is true about the chemistThis statement is true about the chemist’’s response. The s response. The chemist does not establish that perfect accuracy is possible. Bchemist does not establish that perfect accuracy is possible. But ut this is not a good criticism of the chemistthis is not a good criticism of the chemist’’s response because it s response because it is entirely beside the point. Establishing that perfect accuracis entirely beside the point. Establishing that perfect accuracy is y is possible would have, if anything, damages the chemistpossible would have, if anything, damages the chemist’’s s position. So the chemistposition. So the chemist’’s response cannot be legitimately s response cannot be legitimately criticized for failing to establish this.criticized for failing to establish this.

Consider answer choice (b): Consider answer choice (b): This criticism goes to the heart of what is wrong with the This criticism goes to the heart of what is wrong with the chemistchemist’’s response. The chemist dismisses the physicists response. The chemist dismisses the physicist’’s s challenge because of the physicistchallenge because of the physicist’’s alleged motives for making s alleged motives for making it and never actually discusses the merits of the challenge itseit and never actually discusses the merits of the challenge itself. lf. It is directed against the person rather than against the positiIt is directed against the person rather than against the position. on. Here, the chemistHere, the chemist’’s response is clearly irrelevant to the s response is clearly irrelevant to the substance of the physicistsubstance of the physicist’’s claim. The argument that the s claim. The argument that the chemist presents seems more like a rhetorical ploy than a chemist presents seems more like a rhetorical ploy than a serious argument. serious argument.

Page 71: Logical Reasoning

Example 2:Example 2:If Blankenship Enterprises has to switch suppliers in If Blankenship Enterprises has to switch suppliers in the middle of a large production run, the company the middle of a large production run, the company will not show a profit for the year. Therefore, if will not show a profit for the year. Therefore, if Blankenship Enterprises in fact turns out to show no Blankenship Enterprises in fact turns out to show no profit for the year, it will also turn out to be true that profit for the year, it will also turn out to be true that the company had to switch suppliers during a large the company had to switch suppliers during a large production run. production run.

Question: Question: The reasoning in the argument is most The reasoning in the argument is most vulnerable to criticism on which one of the following vulnerable to criticism on which one of the following grounds? grounds?

Page 72: Logical Reasoning

So we know, based on the question, that we should be So we know, based on the question, that we should be looking for a problem with the argument. When you looking for a problem with the argument. When you analyze the argument, you can identify the problem if analyze the argument, you can identify the problem if you recognize that there may well be other reasons for you recognize that there may well be other reasons for not showing a profit besides having to switch suppliers not showing a profit besides having to switch suppliers in the middle of a large production run. This points to in the middle of a large production run. This points to a major oversight in the argument. a major oversight in the argument. Now letNow let’’s consider the answer choices:s consider the answer choices:

(a) The argument is a circular argument made up of an (a) The argument is a circular argument made up of an opening claim followed by a conclusion that merely opening claim followed by a conclusion that merely paraphrases that claim.paraphrases that claim.(b) The argument fails to establish that a condition under (b) The argument fails to establish that a condition under which a phenomenon is said to occur is the only condition which a phenomenon is said to occur is the only condition under which the phenomenon occurs.under which the phenomenon occurs.

Page 73: Logical Reasoning

Consider answer choice (a):Consider answer choice (a):(a) gives a general account of an argument flaw, but close (a) gives a general account of an argument flaw, but close inspection shows that the Blankenship argument does not inspection shows that the Blankenship argument does not have this flaw. That argumenthave this flaw. That argument’’s conclusion says something s conclusion says something quite different from what was said in the argumentquite different from what was said in the argument’’s premise. s premise. The conclusion says The conclusion says ““If there is no profit, then there was a If there is no profit, then there was a switch in suppliers.switch in suppliers.”” The premis is superficially similar, but it The premis is superficially similar, but it says says ““If there is a switch in suppliers, then there will be no If there is a switch in suppliers, then there will be no profit.profit.”” Hence, this answer choice is not a legitimate Hence, this answer choice is not a legitimate criticism. criticism.

Consider answer choice (b):Consider answer choice (b):This is the correct answer. The argument could only succeed This is the correct answer. The argument could only succeed if it showed that switching suppliers in the middle of a large if it showed that switching suppliers in the middle of a large production run is the only condition under which the production run is the only condition under which the company will show no profit for the year. But the argument company will show no profit for the year. But the argument fails to establish this point. Note that this answer choice fails to establish this point. Note that this answer choice points out what is wrong with this particular argument using points out what is wrong with this particular argument using general terms that could cover many different arguments. general terms that could cover many different arguments.

Page 74: Logical Reasoning

Flaws in Arguments: Flaws in Arguments: Matching Argument FlawsMatching Argument Flaws

You may also come across questions like the You may also come across questions like the following: following:

Which one of the following arguments is most Which one of the following arguments is most similar in its flawed reasoning to the argument similar in its flawed reasoning to the argument above? above?

First, try to determine in what way the reasoning First, try to determine in what way the reasoning is flawed. Then go over the arguments in the is flawed. Then go over the arguments in the answer choices until you find the one whose answer choices until you find the one whose reasoning is flawed in just the same way. reasoning is flawed in just the same way.

Page 75: Logical Reasoning

Example:Example:Consider the following argument: Consider the following argument:

If the majority of the residents of the apartment complex If the majority of the residents of the apartment complex complain that their apartments are infested with ants, then complain that their apartments are infested with ants, then the management of the complex will have to engage the the management of the complex will have to engage the services of an exterminator. But the majority of the services of an exterminator. But the majority of the residents of the complex indicate that their apartments are residents of the complex indicate that their apartments are virtually free of ants. Therefore, the management of the virtually free of ants. Therefore, the management of the complex will not have to engage the services of an complex will not have to engage the services of an exterminator.exterminator.

Question: Question: Which one of the following arguments contains a Which one of the following arguments contains a flawed pattern of reasoning parallel to that contained in the flawed pattern of reasoning parallel to that contained in the argument above?argument above?

Page 76: Logical Reasoning

Notice that the question directs us to look for a flawed patternNotice that the question directs us to look for a flawed patternof reasoning in the reference argument and to look for an answerof reasoning in the reference argument and to look for an answerchoice that contains a similarly flawed pattern. So what exactlchoice that contains a similarly flawed pattern. So what exactly is y is the flaw in the reference argument? One of the argumentthe flaw in the reference argument? One of the argument’’s s premises says that under a certain condition the exterminator wipremises says that under a certain condition the exterminator will ll have to come, and the second premise says that this condition ishave to come, and the second premise says that this condition isnot met. The argument concludes that the exterminator will not not met. The argument concludes that the exterminator will not have to be hired. But whathave to be hired. But what’’s wrong with this? The problem is s wrong with this? The problem is that there may be other conditions under which the exterminator that there may be other conditions under which the exterminator has to be hired. For example, there could have been a rodent has to be hired. For example, there could have been a rodent infestation, forcing the manager to call the exterminator. So tinfestation, forcing the manager to call the exterminator. So the he fact that the condition about ants is not met is not a good fact that the condition about ants is not met is not a good enough reason for concluding that the exterminator will not haveenough reason for concluding that the exterminator will not haveto be hired. Without offering any reasons for doing so, the to be hired. Without offering any reasons for doing so, the argument treats one circumstance that would produce a certain argument treats one circumstance that would produce a certain result as though it were the only circumstance under which this result as though it were the only circumstance under which this result comes about. result comes about.

Page 77: Logical Reasoning

LetLet’’s consider the answer choices: s consider the answer choices: (a) The number of flights operated by the airlines (a) The number of flights operated by the airlines cannot be reduced unless the airlines can collect cannot be reduced unless the airlines can collect higher airfares. But people will not pay higher higher airfares. But people will not pay higher airfares, so it is not the case that the number of airfares, so it is not the case that the number of flights will be reduced. flights will be reduced. (b) Most employess will attend the company picnic if (b) Most employess will attend the company picnic if the entertainment committee is successful in getting the entertainment committee is successful in getting a certain band to play at the picnic. But that band a certain band to play at the picnic. But that band will be out of the country on the day of the picnic, will be out of the country on the day of the picnic, so it is not true that most employees will attend. so it is not true that most employees will attend.

Page 78: Logical Reasoning

Considering answer choice (a):Considering answer choice (a):This argument is like the reference argument in that one of its This argument is like the reference argument in that one of its premises asserts that under a certain condition (airlines cannotpremises asserts that under a certain condition (airlines cannotcollect higher airfares) something will happen (schedules will ncollect higher airfares) something will happen (schedules will not be ot be cut). But this argument is unlike the reference argument in thacut). But this argument is unlike the reference argument in that its t its second premise actually meets the condition set out in the firstsecond premise actually meets the condition set out in the firstpremise. We are told that premise. We are told that ““people will not pay higher airfares,people will not pay higher airfares,”” so it so it stands to reason that airlines cannot collect higher airfares. stands to reason that airlines cannot collect higher airfares. And And thus the conclusion thus the conclusion –– that the number of flights will not be reduced that the number of flights will not be reduced –– follows from these premises. So this argument does not exhibitfollows from these premises. So this argument does not exhibitthe same pattern of flawed reasoning as the reference argument. the same pattern of flawed reasoning as the reference argument. In In fact, it does not exhibit flawed reasoning at all. fact, it does not exhibit flawed reasoning at all.

Consider answer choice (b): Consider answer choice (b): Here, too, there is a conditional statement: if a certain band pHere, too, there is a conditional statement: if a certain band plays at lays at the picnic, most employees will attend. So again, under a certathe picnic, most employees will attend. So again, under a certain in condition, something will happen. In this argument, the second condition, something will happen. In this argument, the second premise indicates that the condition will not be met. The band,premise indicates that the condition will not be met. The band,being out of the country, will certainly not play at the picnic.being out of the country, will certainly not play at the picnic. The The argument goes on to conclude that most employees wonargument goes on to conclude that most employees won’’t attend t attend the picnic. This is an exact match of the pattern of reasoning the picnic. This is an exact match of the pattern of reasoning in the in the reference argument, hence it is an exact match of the flaw. reference argument, hence it is an exact match of the flaw.

Page 79: Logical Reasoning

ExplanationsExplanations

Some of the questions in the Logical Reasoning Some of the questions in the Logical Reasoning section require you to identify a potential section require you to identify a potential explanation for some state of affairs that is explanation for some state of affairs that is described to you. Such questions will look like described to you. Such questions will look like these:these:

Which one of the following, if true, most helps to Which one of the following, if true, most helps to explain the difference in average weights?explain the difference in average weights?Which one of the following, if true, most helps to Which one of the following, if true, most helps to resolve the apparent discrepancy in the information resolve the apparent discrepancy in the information given above?given above?

Page 80: Logical Reasoning

Example 1:Example 1:Consider the following:Consider the following:

The company that produces XYZ, a computer spreadsheet program, The company that produces XYZ, a computer spreadsheet program, estimates that millions of illegally reproduced copies of XYZ arestimates that millions of illegally reproduced copies of XYZ are being used. e being used. If legally purchased, this number of copies would have generatedIf legally purchased, this number of copies would have generated millions of millions of dollars in sales for the company, yet despite a companydollars in sales for the company, yet despite a company--wide effort to boost wide effort to boost sales, the company has not taken available legal measures to prosales, the company has not taken available legal measures to prosecute those secute those who have copied the program illegally.who have copied the program illegally.

Question: Question: Which one of the following, if true, most helps to explain why tWhich one of the following, if true, most helps to explain why the company has not taken he company has not taken available legal measures?available legal measures?

Answer Choices:Answer Choices:(a) Many people who purchase a software program like XYZ are wil(a) Many people who purchase a software program like XYZ are willing to ling to purchase that program only after they have already used it.purchase that program only after they have already used it.(b) XYZ is very difficult to copy illegally, because a sophistic(b) XYZ is very difficult to copy illegally, because a sophisticated anticopying ated anticopying mechanism in the program must first be disabled.mechanism in the program must first be disabled.

Page 81: Logical Reasoning

Consider answer choice (a):Consider answer choice (a):This does suggest a reason for the company to tolerate the This does suggest a reason for the company to tolerate the use of illegal copies of its program: those copies happen to use of illegal copies of its program: those copies happen to serve as effective marketing aids in many cases and lead to serve as effective marketing aids in many cases and lead to legal sales of the program. The company may think that it legal sales of the program. The company may think that it has more to lose than to gain from going to court in order to has more to lose than to gain from going to court in order to stop the illegal copying. At the very least, the correct answerstop the illegal copying. At the very least, the correct answertells us that there is a disadvantage for the company in tells us that there is a disadvantage for the company in stopping the illegal copying, and this helps to explain why no stopping the illegal copying, and this helps to explain why no legal measures are taken. legal measures are taken.

Consider answer choice (b):Consider answer choice (b):This does nothing to help us understand the companyThis does nothing to help us understand the company’’s s decision. It may, however, be relevant to some aspect of the decision. It may, however, be relevant to some aspect of the situation. For example, this answer choice does suggest that situation. For example, this answer choice does suggest that those who do the illegal copying are knowledgeable about those who do the illegal copying are knowledgeable about computers and computer software, but it doesncomputers and computer software, but it doesn’’t throw any t throw any light on the companylight on the company’’s decision not to prosecute. s decision not to prosecute.

Page 82: Logical Reasoning

Example 2:Example 2:Consider the following:Consider the following:

Of the five bill collectors at Apex Collection Agency, Mr. Of the five bill collectors at Apex Collection Agency, Mr. Young has the highest rate of unsuccessful collections. Young has the highest rate of unsuccessful collections. Yet Mr. Young is the best bill collector on the agencyYet Mr. Young is the best bill collector on the agency’’s s staff.staff.

Question:Question:Which one of the following, if true, most helps to resolve Which one of the following, if true, most helps to resolve the apparent discrepancy?the apparent discrepancy?

Answer Choices:Answer Choices:(a) Mr. Young(a) Mr. Young’’s rate of collections per year has remained s rate of collections per year has remained fairly steady in the last few years.fairly steady in the last few years.(b) Mr. Young is assigned the majority of the most (b) Mr. Young is assigned the majority of the most difficult cases at the agency. difficult cases at the agency.

Page 83: Logical Reasoning

Consider answer choice (a):Consider answer choice (a):This gives us information that is pertinent to Mr. This gives us information that is pertinent to Mr. YoungYoung’’s performance as a bill collector. But it gives s performance as a bill collector. But it gives us no reason to think that Mr. Young could be the us no reason to think that Mr. Young could be the best bill collector at the agency despite having the best bill collector at the agency despite having the lowest collection rate. lowest collection rate.

Consider answer choice (b):Consider answer choice (b):This gives us reason to think more highly of Mr. This gives us reason to think more highly of Mr. YoungYoung’’s ability as a bill collector, because it makes s ability as a bill collector, because it makes sense to assign the most difficult cases to Mr. Young sense to assign the most difficult cases to Mr. Young if he is very good at collecting bills. And if his rate if he is very good at collecting bills. And if his rate of success is relatively low, this is not really a of success is relatively low, this is not really a surprise, because his cases tend to be more difficult. surprise, because his cases tend to be more difficult.